LSAT and Law School Admissions Forum

Get expert LSAT preparation and law school admissions advice from PowerScore Test Preparation.

 ebertasi
  • Posts: 22
  • Joined: May 28, 2012
|
#4217
Hi,

I narrowed this down to answer choices A and C and the both seem to fill the gap that the author makes about the parallel lines. The only reasoning I can come up with would be that C is to far reaching that would be the answer to possibly a justify question rather than an assumption in this case? However, when I negated both answer choices the both seemed to destroy the argument. What am I missing here?

Thanks for your help!
 Jon Denning
PowerScore Staff
  • PowerScore Staff
  • Posts: 904
  • Joined: Apr 11, 2011
|
#4236
That's a good question, and a subtle distinction between A and C. Look again at the stimulus though, and see which one matches the system described. (A) uses the phrase "the non-Euclidian system of geometry that has the most empirical verification," while (C) says "every non-Euclidian system of geometry that has any empirical verification."

Compare those two to the specifics of the stimulus and suddenly it's clear that (A) is a PERFECT match to the system described by the physicists of the conclusion (the one with the most verification), while (C) is talking about any/every system with any verification. That difference is more than enough to discount (C).

I hope that helps!
 ebertasi
  • Posts: 22
  • Joined: May 28, 2012
|
#4239
Thank you. I did notice the difference between A being specific and C saying that "every...any," however, "every...any" includes the most empirically verified non-Euclidean system of geometry so that's why I saw it as filling the gap. I probably should have gone with A looking back at it because I did notice that. What if A said something different and completely irrelevant. Would C have been a correct answer for an assumption question with that far-reaching language?

Sorry to drag this out. I know that in this case A was a far better choice, but I am just interested to know exactly if C was wrong just because A was better or if C was wrong because of the language and could simply never answer this assumption question correctly.

Thanks again!
User avatar
 Dave Killoran
PowerScore Staff
  • PowerScore Staff
  • Posts: 5853
  • Joined: Mar 25, 2011
|
#4245
Hey Elliot,

Let me jump in for a second since I know Jon is out of the office at the moment and probably won't be able to respond until Monday. There's a couple of points relevant to your question I'd like to address.

Could an answer with "every...any" ever be correct in an Assumption question? Sure, you could easily have an answer with strong language like (C) be correct in the right circumstance. The circumstances here, however, aren't right for an answer this broad and far-reaching.

Let me move next to a comment you made, that you felt (A) "includes the most empirically verified non-Euclidean system of geometry so that's why I saw it as filling the gap." This is an interesting point, and shows the difference between Justify and Assumption questions (and I know Justify questions have been giving you trouble, so this may help there). Think about Assumptions as minimalist elements. The correct answer will be very direct, and not include any extraneous elements. So, if there's anything "extra" in an Assumption answer, that answer will be wrong. In considering (C), you thought that it included the assumption needed for the argument (and that implies you thought it was in there along with other stuff). That's fine, but if an Assumption answer includes anything extra (and (C) does), then it's automatically wrong in an Assumption question. The correct answer is literally the bare bones that are needed, and not a speck extra.

Justify questions are entirely different in that respect. You can have "extra" stuff in a Justify answer and it can still be correct. You just need to meet the minimum threshold to justify the conclusion; anything else that's included doesn't take away from meeting that threshold.

So, really, you were close in your idea of how (C) was working here, but it looks like your conception of what you were looking for may have thrown you off.

Please let me know if that helps. Thanks!
 ebertasi
  • Posts: 22
  • Joined: May 28, 2012
|
#4248
Hey Dave,

That helps a lot. In fact, your clarification reminded me of the virtual module I had watched with the explanation of the apples for assumption and justify questions. How if the answer was 2 apples, an orange, and a pair, then that would be incorrect as an assumption but correct as a justify. The extraneous information made it wrong even though it included 2 apples which was what it needed to fill the gap. This was a very important piece of information that I had apparently forgotten.

Thanks again for the explanation!

Get the most out of your LSAT Prep Plus subscription.

Analyze and track your performance with our Testing and Analytics Package.